LSAT and Law School Admissions Forum

Get expert LSAT preparation and law school admissions advice from PowerScore Test Preparation.

 Administrator
PowerScore Staff
  • PowerScore Staff
  • Posts: 8916
  • Joined: Feb 02, 2011
|
#23849
Complete Question Explanation

Resolve the Paradox. The correct answer choice is (C)

As the two separate estimates of the stock of cod have always matched up in the past, it would be expected that those estimates would continue to match up. As the estimate based upon commercial tonnage has been increasing and the estimate based on sampling has been decreasing, some intervening factor must have come into play that makes at least one of the two estimates less accurate. Therefore, we must find the answer choice the presents the cause for either the increase in the commercial tonnage estimate or the decrease in the sampling estimate.

Answer choice (A): If fishing vessels were underreporting the number of tons of cod they catch, you would expect that to lead to a decrease in the estimate based upon commercial tonnage, not an increase.

Answer choice (B): This answer choice would seem to imply that perhaps that surveying estimate has become more accurate, but it would do nothing to explain why specifically the surveying estimate increased while the commercial estimate decreased.

Answer choice (C): This is the correct answer choice. The commercial tonnage estimate is based upon how much cod was caught by commercial vessels per unit of effort. This estimate would remain accurate so long as the amount of effort to catch fish stayed constant. This answer choice, however, tells us that it does not. Therefore, the increase in the amount of fish caught per unit of effort did not result from an increase in the amount of cod available; it came from the increased efficiency of the commercial fishermen.

Answer choice (D): Same as Answer Choice B, this answer choice may explain why there would be some fluctuation may occur between the commercial and surveying estimates. It does nothing, however, to explain why those estimates are going in the specific directions they are going; nor does it explain why they have only now begun to deviate from each other.

Answer choice (E): Same as Answer Choice A, this answer choice would seem to imply that there would be a decrease in the estimate based upon commercial tonnage, not an increase.
 Jkjones3789
  • Posts: 89
  • Joined: Mar 12, 2014
|
#14731
Hello, When doing this question I wanted to pick C because I knew that the more efficient means of acquiring fish would increase the average .... But ... how does that answer address why the # of cod caught by research vessels has decreased. I understand that the # of cod caught by research vessels is an actual number and the other is an average but I still didn't want to choose C so I chose D instead since those were the only two I had in contention. Thank you :-D
 BethRibet
PowerScore Staff
  • PowerScore Staff
  • Posts: 200
  • Joined: Oct 17, 2012
|
#14735
Hi JK,

Thanks for the question.
Though C does not reference research vessels, it still provides an explanation for the disparity between the two estimates. If the commercial vessels are more efficient, that could explain the increased catch, despite the fact that the research estimate indicates fewer cod are actually present.

The significance of answer choice D is unclear. To help explain, we would have to know more about the difference between the cod available in a 30 day period, as opposed to a continuous period. As it is, it doesn't really tell us anything that helps explain the difference between the two estimates.

Hope that helps!

Beth

Get the most out of your LSAT Prep Plus subscription.

Analyze and track your performance with our Testing and Analytics Package.